Determine the height of the lift, in metres, above the gym floor. show all your work algebraically. round to the nearest cm, if necessary.

Determine The Height Of The Lift, In Metres, Above The Gym Floor. Show All Your Work Algebraically. Round

Answers

Answer 1

Height of lift = x + x = 2x

We can find x using triangle ABC by the cosine rule

[tex]\begin{gathered} x^2=5.6^2+5.6^2-2(5.6)(5.6)\cos40^0 \\ x^2=62.72-48.04352 \\ x^2=14.67648 \\ x=\sqrt{14.67648} \\ x=3.831m \end{gathered}[/tex]

Height of lift = 2 X 3.831m = 7.662m

This will be converted to cm by multiplying by 100

Height of lift = 7.662 X 100 cm

= 766.2 cm

= 766cm ( nearest cm )

Hence the answer is 766cm

Determine The Height Of The Lift, In Metres, Above The Gym Floor. Show All Your Work Algebraically. Round

Related Questions

you are 5 feet and 4 inches tall and cast a shadow 6 feet long. At the same time , a nearby tree cast a shadow 40 feet 6 inches long. Find the heigh of the tree.

Answers

The height of the tree is such that it cast a shadow 40 feet 6 inches long and maintains the given ratio will be 36 feet.

What are the ratio and proportion?

The ratio is the division of the two numbers.

For example, a/b, where a will be the numerator and b will be the denominator.

As per the given question,

The height of a person is 5 feet and 4 inches.

Since 1 feet = 12 inches so 4 inches = 1/3 feet

Therefore,height of a person = 5 + 1/3 = 16/3 feet

Its shadow length = 6 feet

The ratio of the height to shadow length = (16/3)/6

Now, the shadow of the tree = 40 feet and 6 inches so 40.5 feet

Let's say its length was x feet.

Then x/40.5 =  (16/3)/6

x = 36 feet

Hence "The height of the tree is such that it cast a shadow 40 feet 6 inches long and maintains the given ratio will be 36 feet".

For more information about ratios and proportions,

brainly.com/question/26974513

#SPJ1

Find the average rate of change of f(x)=x^2-4x+1 from x=2 to x=6

Answers

Answer:

The answer is 4

Select the correct answer..What is the value of i^ if the remainder of 4 is 2?OA. -i'OB.-1Ос. іOD. 1ResetNext

Answers

1) Considering that for that complex number we have the following pattern:

[tex]\begin{gathered} i^1=i \\ i^2=-1 \\ i^3=-1\cdot i=-i \\ i^4=-1\cdot-1=1 \end{gathered}[/tex]

2) And that, the question asks us about the what number must be that exponent so that the remainder is 2, we can write out:

[tex]\frac{n}{4}=4d+2[/tex]

which d is the divisor, so if the remainder is 2 then we can state:

[tex]i^n=i^2=-1[/tex]

Hello! I need some help with this homework question, please? The question is posted in the image below. Q15

Answers

ANSWER:

A.

[tex]x=-1,-3,11[/tex][tex]f(x)=(x+3)(x-11)(x+1)[/tex]

STEP-BY-STEP EXPLANATION:

We have the following function:

[tex]f(x)=x^3-7x^2-41x-33[/tex]

To find the zeros of the function we must set the function equal to 0 in the following way:

[tex]x^3-7x^2-41x-33=0[/tex]

We reorganize the equation in order to be able to factor and calculate the zeros of the function, like this:

[tex]\begin{gathered} x^3-7x^2-41x-33=0 \\ -7x^2=-8x^2+x^2 \\ -41x=-33x-8x \\ \text{ Therefore:} \\ x^3-8x^2+x^2-33x-8x-33=0 \\ x^3-8x^2-33x=-x^2+8x+33 \\ x(x^2-8x-33)=-(x^2-8x-33) \\ x^2-8x-33 \\ -8x=3x-11x \\ x^2+3x-11x-33 \\ x(x+3)-11(x+3) \\ (x+3)(x-11) \\ \text{ we replacing} \\ x(x+3)(x-11)=-1 \\ x(x+3)(x-11)+(x+3)(x-11)=0 \\ (x+3)(x-11)(x+1)=0 \\ x+3=0\rightarrow x=-3 \\ x-11=0\rightarrow x=11 \\ x+1=0\rightarrow x=-1 \end{gathered}[/tex]

Therefore, the zeros are:

[tex]x=-1,-3,11[/tex]

And in its factored form the expression would be:

[tex]f(x)=(x+3)(x-11)(x+1)[/tex]

What is the value of the expression 2c( a + b) when a = 2, b = 5, and c = 4

Answers

The given expression is

2c(a + b)

From the information given,

a = 2

b = 5

c = 4

By substituting these values into the expression, it becomes

2 * 4(2 + 5)

= 8(7)

= 56

The value of the expression is 56

Line g passes through the points (-2.6,1) and (-1.4.2.5), as shown. Find theequation of the line that passes through (0,-b) and (c,0).

Answers

The blue line passes through the points

(-2.6, 1) and (-1.4, 2.5)

I will label the coordinates as follows for reference:

[tex]x_1=-2.6,y_1=1,x_2=-1.4,y_2=2.5[/tex]

Step 1: Find the slope of the blue line

The slope between two points is calculated with the formula:

[tex]m=\frac{y_2-y_1}{x_2-x_1}[/tex]

We substitute the values and we get that the slope of the blue line is:

[tex]m=\frac{2.5-1}{-1.4-(-2.6)}=\frac{1.5}{1.2}=1.25[/tex]

The slope m of the blue line is 1.25.

step 2: With that slope, calculate b (the intercept of the blue line with the y axis).

For this we use the point - slope equation:

[tex]y=m(x-x_1)+y_1[/tex]

Where we will use the sane x1 and x2 as in the previous step, so we get

[tex]\begin{gathered} y=1.25(x-(-2.6))+1 \\ y=1.25(x+2.6)+1 \\ y=1.25x+3.25+1 \\ y=1.25x+4.25 \end{gathered}[/tex]

We compare this with the slope-intercept equation

[tex]y=mx+b[/tex]

And we can see that the incercept b is 4.25

[tex]b=4.25[/tex]

step 3: Find the value of c.

to find the value of c, we need to know at which point the blue line crosses the x axis.

Since we already have the equation of the blue line y=1.25x+4.25, and the line crosses the x axis at y=0, we substitute this to find the x value that is equal to c:

[tex]\begin{gathered} 0=1.25x+4.25 \\ -4.25=1.25x \\ \frac{-4.25}{1.25}=x \\ -3.4=x \end{gathered}[/tex]

The blue line crosses the x axis at (-3.4,0), thus we can conclude that

[tex]c=-3.4[/tex]

Step 4: Define the two point where the orange line passes through.

We know from the picture that the orange line passes through (c,0) and (0,-b)

Since we have the values of c = -3.4 and b=4.25, we can say that the orange line passes through (-3.4, 0) and (0, -4.25)

Step 5: Calculate the slope of the orange line.

the orange line passes through (-3.4, 0) and (0, -4.25), so we define:

[tex]undefined[/tex]

PLS HELP Rewrite the following equation in slope-intercept form.

y + 8 = –3(x + 7)


Write your answer using integers, proper fractions, and improper fractions in simplest form.

Answers

Answer is y = -3x -29

Step by step

An equation in the slope-intercept form is written as. y=mx+b.
m is the slope of the line and b is the y-intercept.

y +8 = -3x -21

Subtract 8 from both sides to isolate y

y + 8 - 8 = -3x -21 -8

y = -3x -29

Answer:

[tex] \sf \: y = -3x - 29[/tex]

Step-by-step explanation:

Given equation,

→ y + 8 = -3(x + 7)

The slope-intercept form is,

→ y = mx + b

Let's rewrite the equation,

→ y + 8 = -3(x + 7)

→ y + 8 = -3x - 21

→ y = -3x - 21 - 8

→ [ y = -3x - 29 ]

Thus, answer is y = -3x - 29.

Subtracting algorithm

Answers

Using subtracting algorithm, we have 69,000 - 42,547 = 26,453 and 152,681 - 34,820 = 117,861.

According to the question,

We have the following expressions:

_ _ , 0 0 0

-4 2, _ _ _

_________

2 6 , 4 5 3

Now, we will first solve this.

The spaces has to be filed with the digits from 0 to 9  and in both the questions, no digit has to be repeated.

Now, we will solve it from the right hand side.

(Note that we will solve each row that is we will solve vertically.)

0 is given, then we have blank, and in result we have 3.

It is clear that no number can be subtracted from 0. So, we have to regroup it from other digits. So, it will be 10.

Now, 10-3 = 7

So, this blank will be 7.

Now, come to next one.

We have 0 and a blank and then 5 in the result.

Now, it should be 9 because 1 was taken away from it.

So, 9-5 = 4

This blank has to be filled with 4.

Now, we will solve next one.

We have 0, a blank and then 4 in the result.

Now, this 0 will make 9.

9-4 = 5

This blank has to be filled with 5.

Now, we have a blank, 2 and 6 in the result.

1 has been carried from this digit.

So, we have:

6+2+1 = 9

This blank has to be filled with 9.

Next, we have a blank, 4 and 2 in the result.

4+2 = 6

This has to be filled with 6.

Now, we will solve the second question like this.

_ 5 2, 6 8 1

-   _ 4, _ _ _

__________

    1 1 7, 8 6 1

Now, moving from right to left vertically:

We have 1, a blank and 1 in the result.

1-1 = 0

Blank = 0

We have 8, a blank and 6 in the result.

8-6 = 2

Blank = 2

We have 6, a blank and 8 in the result.

Now, 8 can not be subtracted from 6. So, it has to be 16.

16-8 = 8

Blank = 8

Now, next one is complete.

So, we have next one.

We have 5, a blank and 1 in the result.

We will subtract 1 from 4 because 1 was carried away from 5.

4-1 = 3

Blank = 3

Now, we have a blank and 1 in the result.

Blank = 1

Hence, the answers would be  69,000 - 42,547 = 26,453 and 152,681 - 34,820 = 117,861.

To know more about subtracting algorithm here

https://brainly.com/question/12905904

#SPJ1

help me pleaseeeeeeeee

Answers

Answer:

A. 200

B. 500

Step-by-step explanation:

          1000x²

R(x) = --------------

           x² + 4

x = years

A. the first year = 1

         1000(1)²

R(1) = --------------

           (1)² + 4

            1000

R(1) = --------------

             1 + 4

            1000

R(1) = --------------

               5

R(1) = 200

B.  years = 2

     

           1000(2)²

R(2) = --------------

           (2)² + 4

 

           1000(4)

R(2) = --------------

             4 + 4

            4000

R(2) = --------------

                8

R(2) = 500

I hope this helps!

How do I solve it and what would be the answer

Answers

The quotient is x² + 4x + 3

Yes, (x - 2) is a factor of x³ + 2x² - 5x - 6

Explanation:[tex](x^3+2x^2\text{ - 5x - 6) }\div\text{ (x - 2)}[/tex][tex]\begin{gathered} x\text{ - 2 = 0} \\ x\text{ = 2} \\ \\ \text{coefficient of }x^3+2x^2\text{ - 5x - 6:} \\ 1\text{ 2 -5 -6} \\ \\ We\text{ will divide the coefficients by 2} \end{gathered}[/tex]

Using synthetic division:

[tex]\begin{gathered} (x^3+2x^2\text{ - 5x - 6) }\div\text{ (x - 2) = }\frac{(x^3+2x^2\text{ - 5x - 6)}}{\text{(x - 2)}} \\ \frac{(x^3+2x^2\text{ - 5x - 6)}}{\text{(x - 2)}}\text{ = quotient + }\frac{remai\text{ nder}}{\text{divisor}} \\ \\ The\text{ coefficient of the quotient = 1 4 3} \\ \text{The last number is zero, so the remainder = 0} \end{gathered}[/tex][tex]\begin{gathered} \frac{(x^3+2x^2\text{ - 5x - 6)}}{\text{(x - 2)}}=1x^2\text{ + 4x + 3 + }\frac{0}{x\text{ - 2}} \\ \text{quotient }=\text{ }x^2\text{ + 4x + 3} \end{gathered}[/tex]

For a (x - 2) to be a factor of x³ + 2x² - 5x - 6, it will not have a remainder when it is divided.

Since remainder = 0

Yes, (x - 2) is a factor of x³ + 2x² - 5x - 6

The vertices of a rectangle are located at A(4, -1), B(-4, -1), C(-4, 6), and D(4, 6). What is the distance between the side AB and BC respectively?

Answers

The coordinates of the vertices of a rectangle are:

[tex]\begin{gathered} A\left(4,-1\right) \\ B\left(-4,-1\right) \\ C\left(-4,6\right) \\ D\left(4,6\right) \end{gathered}[/tex]

Plotting these points:

The figure shows two right triangles, each with its longest side on the same line. Z 3 X T y 2 R. S 1. Explain how you know the two triangles are similar 2. How long is XY? 3. For each triangle, calculate (vertical side) = (horizontal side). 4. What is the slope of the line? Explain how you know.

Answers

The triangle are similar because the ratio between 2 sides is the same as the ratio between another 2 sides.

The included angle(angle S and Y) are equal( 90 degree)

if you're lonely then be lonely with me but I need your help

Answers

Assuming question is:[tex]-5(7+x)+2\frac{5}{6}x=0[/tex]

Let's use distributive property to simplify:

[tex]-35-5x+2\frac{5}{6}x=0[/tex]

Now we add up the x's and take number to another side:

[tex]\begin{gathered} -5x+2\frac{5}{6}x=35 \\ -2\frac{1}{6}x=35 \end{gathered}[/tex]

We make 2 and one-sixth into improper fraction and divide to get x:

[tex]\begin{gathered} -\frac{13}{6}x=35 \\ x=\frac{35}{-\frac{13}{6}} \\ x=35\cdot-\frac{6}{13}=-\frac{210}{13} \end{gathered}[/tex]

x is "minus 210 over thirteen"

Write an inequality:Carlos was going to sell all of his stamp collection to buy a video game. After selling half of them he changed his mind. He then bought twelve more. How many did he start with if he now has at least 29?

Answers

Answer:

He started with at least 34 stamp collection

Explanation:

Let x represent Carlos' initial stamp collection.

From the question, we're told that he sold half of them, bought twelve more, and currently has at least 29, we can go ahead and set up an inequality as shown below;

[tex]\frac{x}{2}+12\ge29[/tex]

We can go ahead and solve for x following the below steps;

Step 1: Subtract 12 from both sides of the equation;

[tex]\begin{gathered} \frac{x}{2}+12-12\ge29-12 \\ \frac{x}{2}\ge17 \end{gathered}[/tex]

Step 2: Multiply both sides by 2;

[tex]\begin{gathered} \frac{x}{2}\times2\ge17\times2 \\ x\ge34 \end{gathered}[/tex]

From the above, we can say that Carlos started with at least 34 stamp collection

A company plans a major investment and theamount of profit is uncertain, but researchersgive the following estimate for the distribution.1.5210Profit(inmillions)Probability0.10.20.40.20.1What is the expected value of the profit?[?] million dollars

Answers

The expected value is the return you expect from some kind of investment/action.

When we are presented with probabilty of an action, we can take the expected value of the whole table [investment] by taking the sum of the products of probability and the action.

Here, we want products of "probability" and "profit". Then we sum it. Shown below:

[tex]\begin{gathered} E=(0.1)(1)+(0.2)(1.5)+(0.4)(2)+(0.2)(4)+(0.1)(10) \\ E=3 \end{gathered}[/tex]Expected value of profit = 3 million dollars

Given the median QR and trapezoid MNOP, what is the value of X?M3.8033Rکد 73PA. 6B. 19(C. 2D, 5E 7F. Cannot be determined

Answers

SOLUTION

Consider the diagram below

Applying the rule in the diagram above, we have

[tex]|QR|=\frac{1}{2}(|ON|+|PM|)[/tex]

Recall from the questions

[tex]\begin{gathered} |QR|=33 \\ |ON|=3x-8 \\ |PM|=7x+4 \end{gathered}[/tex]

Then we substitute the parameters above into the expression above

[tex]\begin{gathered} 33=\frac{1}{2}(3x-8+7x+4) \\ \text{ Multiply both sides by 2} \\ 66=3x-8+7x+4 \\ \text{rerrange the terms and simplify } \\ 66=10x-4 \\ \text{collect like terms } \\ 66+4=10x \end{gathered}[/tex]

simplify further

[tex]\begin{gathered} 70=10x \\ \text{divide both sides by 10} \\ x=\frac{70}{10} \\ \text{then} \\ x=7 \end{gathered}[/tex]

Therefore the value of x is 7

Therefore the right option is E

The schedule for summer classes is available and Calculus and Introduction to Psychology are scheduled at the same time, so it is impossible for a student to schedule for both courses. The probability a student registers for Calculus is 0.05 and the probability a student registers for psychology is 0.62. What is the probability a student registers for Calculus or psychology?

Answers

Explanation

The given is that the probability a student registers for Calculus is 0.05 and the probability a student registers for psychology is 0.62. Since it impossible for a student to schedule for both courses, we will have

[tex]\begin{gathered} Pr(Psychology\text{ or calculus\rparen=Pr\lparen P\rparen+Pr\lparen C\rparen-Pr\lparen P}\cap C) \\ =0.05+0.62-0 \\ =0.67 \end{gathered}[/tex]

Answer: 0.67

PLEASE ANSWER ASAP1. The length of a bookshelf is 5 ft. The length of a model of this bookshelf is 3 ft. Find the scale of the model to the bookshelf. Enter your answer in the box.2. A rectangular garden has a length of 8 ft and a width of 4 ft. A smaller garden was made, using a scale of 3:4 . Find the dimensions of the smaller garden. Enter your answers in the boxes.

Answers

1.

The scale factor would be:

[tex]3\colon5[/tex]

2.

Divide each original dimension by 4 and then multiply by 3, as following:

[tex]\begin{gathered} \frac{8ft}{4}\cdot3=6ft \\ \\ \frac{4ft}{4}\cdot3=3ft \end{gathered}[/tex]

The smaller garden has a length of 6 ft and a widht of 3 ft

What is the procedure for 31/4-17/8= I'm getting 25/8

Answers

Evaluate the value of the expression:

[tex]3\frac{1}{4}-1\frac{7}{8}[/tex]

Step 1: The mixed fraction will be converted to improper fraction

[tex]\begin{gathered} 3\frac{1}{4}-1\frac{7}{8} \\ =\frac{3\times4+1}{4}-\frac{1\times8+7}{8} \\ =\frac{12+1}{4}-\frac{8+7}{8} \\ =\frac{13}{4}-\frac{15}{8} \end{gathered}[/tex]

Step 2: The denominator of 8 will be used as the L.C.M, since it's the greatest of the two

[tex]\begin{gathered} \frac{13}{4}-\frac{15}{8} \\ =\frac{26-15}{8} \\ =\frac{11}{8} \\ =1\frac{3}{8} \end{gathered}[/tex]

Hence the correct answer = 1 3/8

[tex]1\frac{3}{8}[/tex]

ive tried to do this question multiple times but i just cant seem to understand it

Answers

The domain of the function which is the entire x values during the strike is

[tex]0\leq x\leq230[/tex]

Using the origin as the center of dilation and a scale factor of k=1/2 find the coordinates of the vertices of the image of the polygon below

Answers

Given

Using the scale of 1/2

This means we divide each coordinate by 2

[tex]\begin{gathered} (0,3)=(0,1.5) \\ (-4,0)=(-2,0) \\ (2,0)=(1,0) \\ (0,-3)=(0,-1.5)_{} \end{gathered}[/tex]

The science class is taking a trip to the science center. there are 20 students and an unknown number of adult chaperones going. the bus holds at most 35 people.what is the greatest number of adults who can chaperone? I know the answer is 15. however, what they want us to do is create an equation using inequalities (<, >, etc..) That's what I don't know how to do with this problem. I'm trying to explain it to my daughter but I don't know how to do it myself.

Answers

We are given that there are 20 students and an unknown number of adult chaperones.

Let x denotes the unknown number of adult chaperones.

The bus holds at most 35 people

We know that 20 students plus x number of adult chaperones should be at most 35

at most 35 means equal or less than 35

So we can write

[tex]20+x\le35[/tex]

Now we can easily solve for x

[tex]\begin{gathered} 20+x\le35 \\ x\le35-20 \\ x\le15 \end{gathered}[/tex]

Multiplying Polynomials 5y[tex] ({5y}^{2} + y + 3)(x - 2)[/tex]

Answers

Let's distribute those factors, and expand this:

[tex]\begin{gathered} (5y^2+y+3)\mleft(x-2\mright) \\ 5xy^2-10y^2+xy-2y+3x-6 \\ \end{gathered}[/tex]

There's no further way beyond that. Either we pick the factored form or the expanded version of that polynomial. As this is not an equation we stop it here.

Martin Brothers Moving rents moving vans by the hour. The company charges a flat fee of $17.99, plus an additional $19.99 per hour. One customer pays $57.97 for her rental. How long was her rental?

Answers

Let h be the number of hours that the customer rented the van, then we can set the following equation:

[tex]19.99h+17.99=57.97.[/tex]

Subtracting 17.99 from the above equation we get:

[tex]\begin{gathered} 19.99h+17.99-17.99=57.97-17.99, \\ 19.99h=39.98. \end{gathered}[/tex]

Dividing by 19.99 we get:

[tex]\begin{gathered} \frac{19.99h}{19.99}=\frac{39.98}{19.99}, \\ h=2. \end{gathered}[/tex]

Answer: 2 hours.

Question 10
Answer:
Not yet answered Marked out of 2.00
A tyre made of rubber (density 1.2 g/cm³) has a mass of 3.6 kg.
Find its volume.
(Use ^sign from the computer's keyboard to express the power of the units for the volume
Question 11
Not yet answered Marked out of 1.00
Flag question
What percentage of the grid is shaded?
PFlag question

Answers

1.volume = mass*density

v = 1.2 * 3.6 = 4.32 cm3

The density of a substance quantifies how tightly its molecules are packed, which affects how heavy or light it is.

Density is calculated as follows: density=mass/volume. Most often, mass is measured in grams or kilograms. Most often, volume is measured in cubic centimeters (cm3), cubic meters (m3), or millileters (mL).

How can you find mass from density?

multiply the volume by the density.

Mass per unit volume is the definition of density.

ρ = m V

This can be rearranged to yield the mass expression.

m = ρ × V

Example:

What mass of the liquid is present if 500 mL of it has a density of 1.11 g/mL

m = ρ × V = 500 mL × 1.11 g

1 mL = 555 g

to learn more about density=mass/volume refer to:

https://brainly.com/question/952755

#SPJ13

Question #3 3) The digits of a 2-digit number differ by 5. If the digits are interchanged and the resulting number is added to the original number, we get 99. Find the original number.

Answers

ones number = x

Tens number = y

y>x

Number at the tens place y = (x+5)

original number = 10 (x+5)+x

Interchange digits:= 10x+(x+5)

original number + new number = 99

¨[10(x+5)+x]+ [10x+ (x+5)] =99

Solving for x:

(10x+50+x )+( 10x+x+5) = 99

Combine like terms

(11x+50) + (11x+5) = 99

11x+11x+50+5 =99

22x+55 =99

subtract 55 from both sides

22x +55-55= 99-55

22x = 44

Divide both sides by 22

22x/22= 44/22

x = 2

unit place: 2

tens place = x+5 = 2+5 = 7

original number = 72

What is the probability that a random selected yard will have fewer than 6 trees

Answers

Based on the given histogram on yards and the number of trees they have, the probability that a random selected yard will have fewer than 6 trees is 60%

How to find the probability?

The probability that in a random yard, the number of trees would be less than 6 trees can be found by the formula:

= Proportion of yards with 0 - 2 trees + Proportion of yards with 2 - 4 trees + Proportion of yards with 4 - 6 trees

The probability that a random yard would have fewer than six trees is therefore:

= 0.35 + 0.20 + 0.05

= 0.60

= 60%

Find out more on probability at https://brainly.com/question/26622630

#SPJ1

Permutation/combination In how many ways can five people line up to get on a bus? (Hint: all 5 people are lining up)

Answers

We have to calculate in how many ways can five people line up to get on a bus.

This is a permutation of 5 in 5 without repetition, so it can be calculated as:

[tex]P(5,5)=5!=120[/tex]

Answer: in 120 ways.

triangle OPQ is similar to triangle RST. Find the measure of side RS. Round your answer to the nearest tent if necessary

Answers

To answer this question, we have that, if two triangles are similar, they maintain the same proportion on their corresponding sides.

We have that the corresponding sides are QP and TS, OP and RS, and QO and TR, so we can write:

[tex]\frac{TS}{QP}=\frac{RS}{OP}=\frac{TR}{QO}[/tex]

Then, since we have the values for QP, TS, and OP, we can find RS using the above proportion:

[tex]\frac{TS}{QP}=\frac{RS}{OP}\Rightarrow\frac{41.4}{11}=\frac{RS}{8}\Rightarrow RS=\frac{41.4\cdot8}{11}=\frac{331.2}{11}\Rightarrow RS=30.109090\ldots[/tex]

Then, we have that we can round this value to 30.11 units, and if we round the answer to the nearest tenth, we finally have that RS = 30.1 units.

Answer:

x = 30.1  (round 30)

Step-by-step explanation:

being similar we can solve with a simple equation

11 : 8 = 41.4 : x

x = 8 × 41.4 : 11

x = 331,2 : 11

x = 30.1  (round 30)

5. Noah is solving an equation and one of his moves is unacceptable. Hereare the moves he made. Which answer best explains why the "divide eachside by x step is unacceptable? *2(3+6) - 4= 8 + 6321 + 12 - 4= 8 + 612.1 + 8 = 8 + 6.120 = 602 = 6original equationapply the distributive propertycombine like termssubtract 8 from both sidesdivide each side by IO When you divide both sides of 2x = 6x by x you get 2x^2 = 6x^2.When you divide both sides of 2x = 6x by x it could lead us to think that there is nosolution while in fact the solution is x = 0..aWhen you divide both sides of 2x = 6x by x you get 2 = 6x.aOWhen you divide both sides of 2x = 6x by x it could lead us to think that there is nosolution while in fact the solution is x = 3..

Answers

SOLUTION

Write out the original equation

[tex]2(x+6)-4=8+6x[/tex]

Then, Apply the distributive property on the left hand side of the equation

[tex]\begin{gathered} 2x+12-4=8+6x \\ 2x+8=8+6x \end{gathered}[/tex]

Then combine trhe like terms subtracting 6x from both side

[tex]\begin{gathered} 2x+8-6x=8+6x-6x \\ 2x-6x+8=8 \end{gathered}[/tex]

Subtract 8 from both sides of the last equation

[tex]\begin{gathered} 2x-6x+8-8=8-8 \\ 2x-6x=0 \\ -4x=0 \\ \end{gathered}[/tex]

hence

Divide both sides by -4, we have

[tex]\begin{gathered} -\frac{4x}{-4}=\frac{0}{-4} \\ \text{Then} \\ x=0 \end{gathered}[/tex]

Therefore, the solution is x=0

Hence

When we divide both sides of the equation by x, we have

[tex]\begin{gathered} \frac{2x}{x}=\frac{6x}{x} \\ 2=6 \\ \text{which implies thier is no solution} \end{gathered}[/tex]

While the solution is x=0

Therefore

When we divide the equation by 2x=6x by x it could lead us to think that there is no solution while the solution is x=0

Answer; The second option is right

Other Questions
Hey there Mr or Ms could you help me out here with this problem? Just a head up this isn't a quiz, it's my homework assignment for today it's about Squares and Rhombi. A serum amylase is diluted 1/100 with a result of 45.0 U/L. What is the patient's actual amylase result? H(x)=-5/6x;h(x)=10Find the value of x so that the function has the given value Iuanyware is a ________ application that is designed to run within a computer browser such as firefox, chrome, opera, or edge. Are the graphs of the equations parallel, perpendicular, or neither?x -3y = 6 and x - 3y = 9 Two planes, which are 2320 miles apart, fly toward each other. Their speeds differ by 80 mph. If they pass each other in 4 hours,what is the speed of each?Step 1 of 2: Use the variable x to set up an equation to solve the given problem. Set up the equation, but do not take steps to solve it. determine the area of figure round to the nearest tenth if necessary.. what kind of kinship is most common in the contemporary united states? a. matrilateral kinship b. bilateral kinship c. patrilateral kinship d. collateral kinship e. generational kinship ANWERS ASAP PLSSSS :) HELPPPPAbigail buys 3 gallons of milk a week. How many pints of milk does she buy? Which equation is equivalent to - 2x + 5 - 3x = 5x + 25?A. -5 = -30B. -6x + 5 = 5x + 25C. - 10x = 20D. 20x - 5 = 25 Using the drawing tools to form the correct answer on the provided graph Which property is demonstrated below?a(b+c)=(a.b)+(a.c)O Inverse propertyO Distributive propertyO Communitive propertyO Identity property( and 15 points for this and will make brainliest to best answer) Please be fast! write two minutes speech for declamation on this topics:If failures are a stepping stone, Ive built myself a ladder to heaven The distance between the points (-2,y) and (3, -7) is 13 units.What are the possible values of y? Monica is the cruelest character in it . Identify a pair of lines that looks parallel in the diagram.pls help me The table shows claims and theirprobabilities for an insurancecompany.Amount of claim(to the nearest $20,000)$0$20,000$40,000$60,000$80,000$100,000Probability0.700.160.090.030.010.01 in the diagram below , the measure of arc SQ is 160 degrees and the measure of arc PR is 102 degrees . find the value of x .